You are on page 1of 40

LEGALEDGE TEST SERIES

Part of the Most Awesome and Consistently Successful Study Material and Test Series Module, spanning across
both Physical and Online Programs in the entire Country. While most of the world fumbled and faltered, 2020
has been another inspiring Success story both for us and those who chose to trust us. As a result LE was able to
engineer Clean-Sweep-Landslide figures of a handsome 35 Selections under 100 ranks, and a whopping 180
selections under 500 ranks in CLAT 2020. With AILET being no different, a total of 30 LEtians found their way
into NLUD in 2020. Read on!

MOCK COMMON LAW ADMISSION TEST 2022


MOCK CLAT #06

Candidate Name : _________________


Duration : 120 Minutes
Batch : _________________
Max. Marks : 150
Contact No. : _________________
Centre Name : __________
Date of Exam : _________________

INSTRUCTIONS TO CANDIDATES

1. No clarification on the question paper can be sought. Answer the questions as they are.
2. There are 150multiple choice objective type questions.
3. Each question carries ONE mark. Total marks are 150.
4. There is a negative marking of 0.25 marks for every incorrect answer.
5. Candidates have to indicate the correct answer by darkening one of the four responses provided, with a
BALL PEN (BLUE OR BLACK) in the OMR Answer Sheet.
Example: For the question, "Where is the TajMahal located?", the correct answer is (b).
The candidate has to darken the corresponding circle as indicated below :
(a) Kolkata (b) Agra (c) Bhopal (d) Delhi
Right Method Wrong Methods

6. Answering the questions by any method other than the method indicated above shall be considered
incorrect and no marks will be awarded for the same.
7. More than one response to a question shall be counted as wrong.
8. The candidate shall not write anything on the OMR Answer Sheet other than the details required and in
the spaces provided for.
9. After the Test is over, the candidate has to return the OMR Answer Sheet to the invigilator. The
candidate should take the Test Paper along with them.
10. The use of any unfair means by any candidate shall result in the cancellation of his/her candidature.
11. Impersonation is an offence and the candidate, apart from disqualification, may have to face criminal
prosecution.
12. Electronic gadgets like mobile phones, pagers or calculators are strictly not permitted inside the Test
Centre/Hall.
13. The candidates shall not leave the hall before the Test is over.
SECTION-A : ENGLISH LANGUAGE

Directions (Q.1-Q.30): Read the passages carefully and answer the questions.

Passage (Q.1-Q.5): On Monday, Assam and Mizoram clashed over a disputed boundary. Five policemen
and one civilian (all from Assam) were killed and nearly 50 others were injured. The bloodiest-ever
incident between the two states was the culmination of the discontent that had been simmering since
October 2020 over incidents of violence and Assam’s residents blocking NH-306, the lifeline to Mizoram,
for 12 days. While Assam claims Mizoram has encroached on its boundary, Mizoram maintains the area
belongs to the state.
Mizoram has accused Assam police personnel of entering its territory, indulging in violence, and claimed
that it only retaliated after tear gas canisters and grenades were hurled by Assam’s security forces.
Assam has blamed Mizoram of breaching existing agreements and status quo on the border, and
accused the Mizoram police of firing on its security forces and civilians using light machine guns. A
violent clash of this nature between two Indian states represents a constitutional breakdown. And the fact
that the clash played out on Twitter, with chief ministers (CMs) of both states sparring and urging home
minister Amit Shah to intervene, represents a political and institutional failure. Incidentally, Mr Shah had
met the CMs of all Northeastern states over the weekend in Shillong where long pending inter-state
border disputes were discussed.
Mizoram’s border dispute with Assam goes back to 1972 when the former was first carved out as a Union
Territory (it became a state in 1987). Assam also has border disputes with Meghalaya, Nagaland and
Arunachal Pradesh. With states in the region either ruled by Bharatiya Janata Party (BJP) governments
or, as in the case of Mizoram, by constituents of the North East Democratic Alliance (NEDA), a BJP-led
political platform of anti-Congress parties, there were expectations that border issues would get resolved.
Instead, the situation appears to have become worse. Monday must serve as a wake-up call for the
Centre as well as states in the region.

1. According to the passage which of the following options is true?


(a) In Mizoram, the government is of Indian National Congress.
(b) CMs of both the states had asked Amit Shah to intervene in the dispute.
(c) Recently Assam and Manipur clashed over boundary dispute causing death of Assam police
personnel.
(d) Due to said clash between the two states, 5 police personnel have died who belonged to Manipur.

2. Why did author expect that the border issues would get resolved in the North-eastern region?
(a) Because recently, there has been cooperation and peace in the north-eastern region.
(b) Because the border dispute was resolved in 1972.
(c) Because all the states in the North easter regions are ruled either by BJP or its alliances.
(d) None of the above.

3. Which of the following can be a suitable title for the passage ?


(a) Assam’s encroachment on Mizoram’s land.
(b) North-eastern States of India – A Disturbed Region
(c) Clash between Mizoram and Assam – An Institutional Failure.
(d) Northeast States of India

4. Which of the following means the same as ‘simmering’ as it is used in the passage?
(a) carping
(b) tricking
(c) stewing
(d) bickering

Head Office: 127, Zone II, MP Nagar, Bhopal |+91-7676564400| https://www.toprankers.com Page 2 of 40
5. Which of the following CANNOT be used as a replacement for ‘culmination’ as it is used in the passage?
(a) nadir
(b) pinnacle
(c) apogee
(d) apotheosis

Passage (Q.6-Q.10): It has been eight working days since the monsoon session of Parliament began. It
started out with the hope that both the treasury and Opposition benches have a common interest in a
functional session. The government wanted to push through its legislative agenda and was prepared to
address the issue of the management of the second wave of Covid-19 this summer. The Opposition saw
an opportunity to hold the government accountable on public health, price rise and national security. But
with the Pegasus revelations storming the Indian public sphere, the script changed. The Opposition has
now demanded that unless there is a full discussion in both Houses on Pegasus and an enquiry, it won’t
allow Parliament to run. The government has issued two perfunctory statements on the issue but
doesn’t want a discussion. And so there is a stalemate.
For the sake of India’s parliamentary democracy, it is crucial to break the stalemate. In principle, the
Opposition is right. The fact that the numbers of key political figures, business executives, bureaucrats,
activists and journalists were on a potential surveillance list, and some of their phone devices were found
to be infected with Pegasus — a military-grade spyware — is cause for concern. It throws up questions
about who was culpable, what it means for the right to privacy, liberty, dignity, and indeed, the very idea
of democracy. Parliament is the best forum to discuss the issue — and it is indeed beyond the purview of
just the information technology minister to address it.
The government is in a bind. It cannot admit to procuring and using Pegasus — given that hacking is
illegal in India and it will raise difficult questions about its democratic credentials. It cannot deny using
Pegasus — given that this means a foreign government was possibly deploying it against Indian citizens.
And so it has relied on a defence based on how all interception and surveillance in India has to be legally
sanctioned. But the fact is that at some point, if not in the House, then in the courts where a bunch of
petitions have been lined up, the government will have to give a detailed response. The government may
think that the disruption is costing it little; it is still able to get legislative business through amid the din
and is able to evade more uncomfortable electoral issues such as price rise. But the spirit of democracy
dictates that the Opposition must be allowed its say, even if the government has its way.

6. According to the passage, why is Government “in a bind”?


(a) It cannot admit that it was using Pegasus because hacking is illegal in India.
(b) It cannot deny using Pegasus because it would mean foreign government was using against Indian.
(c) Both a) and b)
(d) None of the above.

7. Which of the following can be s suitable tone of the passage?


(a) Descriptive
(b) Narrative
(c) Analytical
(d) None of the above.

8. According to the passage, which of the following is not true?


(a) Pegasus is a military grade software.
(b) A number of political figures, business executives, bureaucrats, activists and journalists were on a
potential surveillance list of Pegasus.
(c) It has been eight working days since the Winter session of Parliament began
(d) The Opposition has now demanded that unless there is a full discussion in both Houses on Pegasus.

Head Office: 127, Zone II, MP Nagar, Bhopal |+91-7676564400| https://www.toprankers.com Page 3 of 40
9. Which of the following DOES NOT mean the same as ‘perfunctory’ as it is used in the passage?
(a) desultory
(b) peremptory
(c) cursory
(d) uninterested

10. Which of the following means the same as ‘din’ as it is used in the passage?
(a) commotion
(b) business
(c) session
(d) None of the above.

Passage (Q.11-Q.15): When BS Yediyurappa resigned as Karnataka’s chief minister (CM) on Monday
morning, it truly marked the end of an era for both the Bharatiya Janata Party (BJP) and the state. For the
party, Mr Yediyurappa was the last man standing from the old guard who had retained a position of
power in the formal governance structure. Now 78, he joined the Rashtriya Swayamsevak Sangh over
five decades ago and rose up the Jana Sangh and the BJP hierarchy — all the way from a district to
state unit president; from a member of the legislative assembly first elected way back in 1983 to a
member of Parliament elected in 2014 (the short national stint was only because he wanted to stay
focused on the state); from a three-term leader of opposition in the assembly to a four-term chief minister,
though two of those terms were of less than a week’s duration. But beyond the formal positions, it was Mr
Yediyurappa’s blood, sweat, and tears that enabled the BJP to cross the rubicon and become a truly
national party with an imprint in the south.
In the state, Mr Yediyurappa draws his power from the Lingayat community, which constitutes over 15%
of the population and with a network of sub-religious institutions, wields disproportionate influence over
politics. But the Karnataka satrap also went beyond his caste base to expand the BJP’s footprint among
other communities. This multi-caste alliance has been a strength, but has also generated tensions as is
visible in the battle for succession between leaders of Lingayat and non-Lingayat communities in the
state. Mr Yediyurappa’s administrative record is hard to judge — for he never fulfilled a full term in office.
But perhaps in a symbol of the close nexus that begun to mark the nature of India’s compromised crony
capitalism in the 2000s, he faced corruption allegations on land and mining issues, most infamously due
to his perceived links with the mining barons of Bellary. This eroded the party’s image as one with a
difference.
The BJP now has to ensure a smooth leadership transition, keeping all factions and castes happy. It also
has to ensure that the exit doesn’t have implications for the stability of the government. Memories of its
electoral setback when Mr Yediyurappa briefly left the party between 2012 and 2014 will be fresh in the
BJP’s mind. And it has to ensure that Mr Yediyurappa does not cash a long shadow over his successor.
How it navigates these challenges will determine the future of the party in the state.

11. According to the passage, which of the following is NOT TRUE about YS Yediyurappa?
(a) He got first elected to Legislative Assembly in 1982.
(b) Mr Yediyurappa draws his power from the Lingayat community, which constitutes over 15% of the
population.
(c) He recently resigned as the CM of Karnataka.
(d) None of the above.

12. According to the author, why is YS Yediyurappa’s political career ‘difficult to judge”?
(a) He had lost a huge number of elections before winning one.
(b) He was hated by some people and loved by others.
(c) He never completed a term in the office.
(d) None of the above.

Head Office: 127, Zone II, MP Nagar, Bhopal |+91-7676564400| https://www.toprankers.com Page 4 of 40
13. What was YS Yediyurappa biggest contribution towards BJP?
(a) He helped BJP reach South India.
(b) He helped vote bank of BJP by including Muslim communities in it.
(c) Both a) and b)
(d) Neither a) nor b)

14. It is written in the passage - it was Mr Yediyurappa’s blood, sweat, and tears that enabled the BJP to
cross the rubicon and become a truly national party with an imprint in the south. What is the meaning of
the phrase “cross the rubicon”?
(a) To become significant in your sphere of work.
(b) To reach a point where you cannot change a decision or course of action.
(c) To be most powerful in a region.
(d) To be able control everything.

15. Which of the following means the same as ‘crony’ as it is used in the passage?
(a) dubious
(b) intimate
(c) sacrilegious
(d) vexatious

Passage (Q.16-Q.20): A month ago, both the Bharatiya Janata Party (BJP) and the Congress were
struggling with the political situation in two states where they were the governing party. In Uttar Pradesh,
the management of the second wave triggered criticism of chief minister (CM) Yogi Adityanath; there was
also criticism of his governance style and alleged patronage to certain castes. The BJP national
leadership let the controversy play out, uncharacteristically, in public view. But eventually, it backed Mr
Adityanath’s continuation as CM while making some adjustments, both at the central and state level, to
accommodate leaders of other castes. With this, it hopes to tap into both the incumbency vote (in favour
of the CM) as well as the vote of the discontented (who may be disenchanted with the CM but have faith
in the prime minister).
In Punjab, the acrimonious battle between CM Captain Amarinder Singh and rebel Navjot Singh Sidhu
played out, with the national leadership allowing Mr Sidhu to undermine the CM. Eventually, a power-
sharing formula was imposed with Mr Sidhu appointed as party chief. With this, the Congress too hopes
to tap into both the pro-incumbency vote (of supporters of Captain Singh) and the anti-incumbency vote
(with Mr Sidhu doubling up both as party chief but also chief dissenter against the state government).
While the context is different, the BJP’s formula is cleaner to execute — back the CM, while ensuring the
PM is seen as above the fray. The Congress’s formula is harder to execute — since the CM has to deal
with an official rival in the same party structure.
But both national parties have turned their focus now to two other states. For the BJP, it appears a
transition in Karnataka is imminent, with BS Yediyurappa being asked to make way for another leader as
CM. The veteran Karnataka leader has an autonomous base, especially within the Lingayat community,
and cannot be brushed aside easily. How his possible exit impacts the party’s fortunes in the south is to
be seen. For the Congress, the next challenge is Rajasthan, where Sachin Pilot has been waiting for
promises to be implemented. The party leadership is now seeking to ensure the accommodating of his
loyalists in the state power structure, but whether Mr Pilot’s rival, CM Ashok Gehlot, will allow this
remains unclear. From UP to Punjab, Karnataka to Rajasthan, both national parties are attempting to
manage state units with an eye on the next election and generational transition, while ensuring Delhi’s
writ runs.

16. According to the passage, which of the following options is NOT TRUE regarding the situation in Punjab?
(a) The governing party in Punjab is BJP.
(b) CM of Punjab is Navjot Singh Sidhu.
(c) Mr Amarinder Singh has been appointed as the party chief.
(d) None of the above.

Head Office: 127, Zone II, MP Nagar, Bhopal |+91-7676564400| https://www.toprankers.com Page 5 of 40
17. According to the passage, why did BJP include members of certain castes at Centre and State level?
(a) To accumulate the votes of those castes.
(b) Because those members are highly skilled at specific ministerial work
(c) Because the incumbent ministers were not working well.
(d) None of the above

18. Which of the following can be a suitable title for the passage?
(a) Battle between Sidhu and Amarinder Singh in Punjab
(b) Entry of New Ministers in Central Government
(c) Political Adjustments made by BJP and Congress
(d) None of the above.

19. Which of the following can be used as a replacement for ‘acrimonious’ as it iss used in the passage?
(a) pectoral
(b) lowly
(c) modicum
(d) rancorous

20. Which of the following does NOT MEAN the same as ‘imminent’ as it is used in the passage?
(a) miscible
(b) approaching
(c) impending
(d) forthcoming

Passage (Q.21-Q.25): Mirabai Chanu’s silver medal at the Tokyo Olympics was not unexpected, but it
was a joyous moment when it happened nonetheless. Chanu, who comes from a farming village near
Imphal, has been one of the world’s best weightlifters in her category, 49kg, for some time now. She has
won a world championship and holds the world record in one of the Olympics lifts, the clean & jerk. The
Olympic silver simply adds to the aura and signals, yet again, the pioneering path forged by athletes
from Manipur.
Kunjarani Devi, the woman who inspired Chanu, was India’s first medallist at the world weightlifting
championship. She won a silver in 1989, at a time when being a woman weightlifter in India was almost
unheard of. Devi inspired, and continues to inspire, generations of aspiring athletes to take up the sport in
India. MC Mary Kom did the same with boxing — opening the way for women to get into a fighting sport
and dream of big medals. Six world titles and an Olympic medal later, she is still fighting, in Tokyo. From
the late Dingko Singh to Sarita Devi, and an assembly line of hockey and football players, Manipur has
an outsized influence on the Indian sporting scene.
Much of that has to do with the love for sport and the sporting culture of the state. Yet, Manipur’s sporting
infrastructure is below par, even compared to the usual less-than-ideal standards everywhere in India.
Some states have, in the past decade, started to tap into sporting culture and encourage its
development. Haryana has done that with boxing and wrestling at the grassroots. Odisha has done so
with hockey and athletics and built world class infrastructure. Karnataka has some of the finest sporting
institutes in India now. Manipur would do well to embrace its unique sporting identity and nurture it in a
truly ambitious manner.

21. Which of the following can be a suitable title for the passage?
(a) Enhancing Manipur’s Sports Infrastructure
(b) Mira Bai Chanu and her Achievements
(c) Weightlifting in Olympics
(d) Increase of Women in Olympic Games

Head Office: 127, Zone II, MP Nagar, Bhopal |+91-7676564400| https://www.toprankers.com Page 6 of 40
22. Which of the following sports athletes in not mentioned in the passage?
(a) Mirabai Chanu
(b) Sarita Devi
(c) Saina Nehwal
(d) MC Mary Kom

23. Which of the following NOT TRUE according to the passage?


(a) Kunjarani Devi, was India’s first medallist at the world weightlifting championship
(b) Mirabai Chanu won a silver medal at the Tokyo Olympics
(c) Mirabai Chanu has won six world titles and an Olympic medal.
(d) Karnataka has some of the finest sporting institutes in India now.

24. Which of the following means the same as ‘pioneering’ as it is used in the passage?
(a) titillating
(b) new
(c) scintillating
(d) victorious

25. Which of the following means the closest to ‘ambitious’ as it is used in the passage?
(a) blatant
(b) forcible
(c) zealot
(d) determined

Passage (Q.26-Q.30): Mahabaleshwar, a hill station in Maharashtra’s Satara district in the Western
Ghats, on Friday, recorded the highest ever rainfall in its history: 60cm in the previous 24 hours. More
than 35 people have died in rain-induced landslides in the state’s coastal Raigad district. Mumbai
continues to be battered by heavy rainfall and has been placed under orange alert (“heavy to very heavy
rain at isolated places”). Other parts of the western coast, including cities and towns in Madhya Pradesh,
Karnataka and Goa, have also recorded exceptional rainfall and flooding in the last 24 hours. The India
Meteorological Department (IMD) classifies rainfall as “extremely heavy” if an area records 20 cm or
more in 24 hours. An offshore trough running from the Maharashtra coast to north Kerala coast is causing
such heavy rains and is likely to weaken from July 26, said IMD.
Over the past month, India has seen several rain-related extreme weather events. Last week, 31 people
died in a series of house collapses after a short burst of intense rain triggered landslides in Mumbai. Both
Uttarakhand and Himachal Pradesh have seen incessant rainfall, with a flash flood in the latter. Delhi has
seen some shorter, intense bursts of rain in the last few days, but only after a slow and delayed start to
the monsoon. These wayward weather events carry not only the indelible sign of a surging climate crisis
but are also a compelling reminder that the country is under-equipped to tackle many of its effects, such
as urban flooding and flash floods. But just blaming the climate crisis will be wrong. Despite some
proactive efforts by some states, India’s development plans are marked by frenzied and unregulated
construction, even on flood plains, an utter disregard for natural topography and hydro-geomorphology,
and poor-quality affordable housing. Along with overburdened drainage, these are only exacerbating the
effects of the climate crisis, leading to large-scale human tragedies and destruction of property.
In 100 days, the world will witness the Cop26 summit, the vital United Nations climate talks that open on
November 1 in Glasgow, United Kingdom. The extreme weather events in India and across the world
(floods across Europe and China, wildfires in the United States, killer heatwaves stretching into northern
latitudes) are a sharp reminder of what is at stake, and the express need to accelerate worldwide
measures that are required to control the climate crisis.

Head Office: 127, Zone II, MP Nagar, Bhopal |+91-7676564400| https://www.toprankers.com Page 7 of 40
26. According to the passage which of the following options is NOT TRUE ?
(a) Mahabaleshwar, a hill station in Maharashtra’s Satara district in the Western Ghats, on Friday,
recorded the highest ever rainfall in its history
(b) In 100 days, the world will witness the Cop24 summit, the vital United Nations climate talks that open
in Glasgow, United Kingdom
(c) Recently, more than 30 people died in a series of house collapses after a short burst of intense rain
triggered landslides in Mumbai.
(d) None of the above.

27. According to the passage which of the following reasons exacerbate the effects of climate crisis in India?
(a) Overpopulation
(b) Unregulated construction
(c) Deforestation
(d) Wastage of water resources

28. Which of the following is not an ‘extreme weather event which’ is mentioned in the passage?
(a) Floods in Europe
(b) Wildfires in USA
(c) Heavy Rainfalls in India
(d) None of the above

29. Which of thew following can be used as a replacement for ‘wayward’ as it is used in the passage?
(a) pivotal
(b) awry
(c) refractory
(d) equitable

30. Which of the following does not mean the same as ‘frenzied’ as it is used in the passage?
(a) quietude
(b) frenetic
(c) turbulent
(d) manic

Head Office: 127, Zone II, MP Nagar, Bhopal |+91-7676564400| https://www.toprankers.com Page 8 of 40
SECTION-B : GENERAL KNOWLEDGE/CURRENT AFFAIRS

Directions (Q.31–Q.67): Read the information given below and answer the questions based on it.

Passage (Q.31-Q.34): A museum dedicated to the country's 5,000-year maritime history will be set up in
Gujarat's [1] by the Culture Ministry in collaboration with the [2], officials said on Wednesday.

The two ministries signed a Memorandum of Understanding for 'Cooperation in Development of National
Maritime Heritage Complex (NMHC) at [1], Gujarat' on Wednesday.

According to a statement from the Culture Ministry, the world-class facility will be developed in the vicinity
of the Archaeological Survey of India (ASI) site in [1], located about 80 km away from Ahmedabad.

NMHC would be developed as an international tourist destination where the maritime heritage of India --
from ancient to modern times -- would be showcased and an edutainment approach using the latest
technology would be adopted to spread awareness about the country's maritime heritage, it said.

The NMHC project has been taken up under the unique and innovative projects category of the
Sagarmala programme under MoSPW. The Government of Gujarat has transferred 375 acres of land in
Saragvada village on lease for 99 years at a token rate to MoSPW.

31. Which of the following will replace [1] in the above passage?
(a) Dholavira
(b) Lothal
(c) Pavagadh
(d) Patan

32. Which of the following recently became the UNESCO’s World heritage Site?
(a) Ahmedabad
(b) Rani ki Vav
(c) Dholavira
(d) Champaner

33. Which of the following is not true about the Indus Valley Civilisation?
(a) It flourished around 2,500 BC, in the western part of South Asia, in contemporary Pakistan and
Western India.
(b) The Indus Valley was home to the largest of the four ancient urban civilizations of Egypt,
Mesopotamia, India and China.
(c) In 1929, John Marshall, Director-General of the ASI, announced the discovery of a new civilisation in
the Indus valley to the world.
(d) In 1920s, the Archaeological Department of India carried out excavations in the Indus valley wherein
the ruins of the two old cities, viz. Mohenjodaro and Harappa were unearthed.

34. Which of the following will replace [2] in the above passage?
(a) Ministry of Home Affairs
(b) Ministry of Education
(c) Ministry of Shipping and Port
(d) Ministry of Environment, Forest and Climate Change

Head Office: 127, Zone II, MP Nagar, Bhopal |+91-7676564400| https://www.toprankers.com Page 9 of 40
Passage (Q.35-Q.38): In the ongoing Monsoon Session of Parliament that ends this Friday, the
government will introduce the [1] Constitution Amendment Bill in Parliament to clarify “some provisions in
the 102nd Constitutional amendment Bill” to restore the power of the states to identify backward classes.

According to the Constitution of India, Articles 15(4), 15(5) and 16(4) confer power on a state to identify
and declare the list of socially and educationally backward classes. As a matter of practice, separate
OBC lists are drawn up by the Centre and each state concerned.

The amendment was necessitated after the Supreme Court in its Maratha reservation ruling in May
upheld the 102nd Constitutional Amendment Act but said the president, based on the recommendations
of the National Commission for Backward Classes (NCBC), would determine which communities would
be included on the state OBC list.

"In the task of identification of SEBCs, the President shall be guided by the Commission set up under
Article 338B; its advice shall also be sought by the state in regard to policies that might be framed by it. If
the commission prepares a report concerning matters of identification, such a report has to be shared
with the state government, which is bound to deal with it, in accordance with provisions of Article 338B.
However, the final determination culminates in the exercise undertaken by the President (i.e. the Central
Government, under Article 342A (1), by reason of Article 367 read with Section 3 (8) (b) General Clauses
Act)," the judgment read.

35. Which of the following will replace [1] in the above passage?
(a) 127th
(b) 129th
(c) 131st
(d) 135th

36. Which of the following commissions recommended the Maratha reservation?


(a) Justice PN Bhagwati Committee
(b) Justice BP Mandal Commission
(c) Justice NG Gaikwad Commission
(d) Justice Khanwilkar Commission

37. Consider the following about the provision related to Amendment in the Constitution of India:
I. There is no provision of joint sittings on a Constitution Amending Bill.
II. A Constitution Amendment Bill can only be introduced in Lok Sabha
Which of the above is/are true?
(a) Only I
(b) Only II
(c) Both I and II
(d) Neither I nor II

38. Which of the following Supreme Court Judgement held the rule of 50% limit on reservations?
(a) Keshwanand Bharti v. Union of India
(b) M. Nagaraj & Others vs Union of India
(c) Indra Sawhney vs Union of India
(d) Jarnail Singh vs Lachhmi Narain Gupta

Head Office: 127, Zone II, MP Nagar, Bhopal |+91-7676564400| https://www.toprankers.com Page 10 of 40
Passage (Q.39-Q.42): Prime Minister Narendra Modi announced Friday the renaming of the Rajiv
Gandhi Khel Ratna Award, the country’s highest sporting honour, after hockey player Major Dhyan
Chand, setting off a political war of words with the Opposition Congress.

The Prime Minister said the “exceptional performance” of the men and women hockey teams in the
Tokyo Olympics will rekindle interest in the game across India.

The Congress welcomed the move, but said the Prime Minister “should not have dragged the name of a
celebrated player like Major Dhyan Chand for his myopic political motives”.

It asked him to rename stadiums across the country, including the Narendra Modi stadium in
Ahmedabad, after sports icons. The party accused the Prime Minister of trying to divert attention from
issues like the Pegasus row, farmer protests and rise in prices.

39. When was the Rajiv Gandhi Khel Ratna Established?


(a) 1985
(b) 1988
(c) 1992
(d) 1996

40. Consider the following:


I. Its first recipient was chess legend Viswanathan Anand.
II. Award comprises of a medallion, a certificate, and a cash prize of ₹25 lakh.
Which of the above is/are true?
(a) Only I
(b) Only II
(c) Both I and II
(d) Neither I nor II

41. Major Dhyan Chand was popularly known as which of the following?
(a) The Hockey Genius
(b) The Hockey Magician
(c) The Hockey Wizard
(d) The Hockey Champion

42. Consider the following About Major Dhyan Chand:


I. He was was part of the Olympic team that won gold medals in 1928, 1932 and 1936.
II. Country’s highest award for lifetime achievement in sports is known as the Dhyan Chand Award.
Which of the above is/are true?
(a) Only I
(b) Only II
(c) Both I and II
(d) Neither I nor II

Head Office: 127, Zone II, MP Nagar, Bhopal |+91-7676564400| https://www.toprankers.com Page 11 of 40
Passage (Q.43-Q.47): A Parliamentary Standing Committee has recommended that India should
renegotiate the Indus Water Treaty (IWT) with Pakistan in view of the “present day pressing issues such
as climate change, global warming and environmental impact assessment”.

Headed by MP Sanjay Jaiswal, the Parliamentary Standing Committee on Water Resources submitted
the 12th report on ‘Flood management in the country including international water treaties in the field of
water resources management with particular reference to treaty/agreement entered into with China,
Pakistan and Bhutan’ in both the Lok Sabha and Rajya Sabha on Thursday.

The Committee observed that although the Indus Water Treaty has stood the test of time, it “was framed
on the basis of knowledge and technology existing at the time of its agreement”, when the perspective of
both the nations at that time was confined to river management and usage of water through the
construction of dams, barrages, canals and hydro-power generation.

In recent times, India has often threatened to stop water from flowing into Pakistan, especially in the
aftermath of terror attacks. But the IWT has stood the test of time and has continued to work despite the
two neighbours engaging in three major wars and numerous skirmishes.

43. Indus Water Treaty was signed in which of the following years?
(a) 1956
(b) 1960
(c) 1968
(d) 1970

44. The Treaty under which the commission is formed is backed by which of the
following?
(a) World Bank
(b) United Nations
(c) European Union
(d) USA

45. The Treaty which distributes the water of Indus and its tributaries was signed by
Indian Prime Minister Nehru and Pakistan’s President _________
(a) Yahya Khan
(b) Zulfikar Ali Bhutto
(c) Iskandar Mirza
(d) Ayyub Khan

46. Consider the following about Parliamentary Committees:


I. Parliamentary committees draw their authority from Article 105 and Article 118.
II. In 1993, 17 Departmentally-related Standing Committees (DRSCs), later increased to 24, were
constituted in the Parliament.
Which of the above is/are true?
(a) Only I
(b) Only II
(c) Both I and II
(d) Neither I nor II

Head Office: 127, Zone II, MP Nagar, Bhopal |+91-7676564400| https://www.toprankers.com Page 12 of 40
47. In independent India, the first Public Accounts Committee was constituted in which of the following
years?
(a) 1950
(b) 1951
(c) 1952
(d) 1954

Passage (Q.48-Q.52): The UN General Assembly approved a resolution Monday establishing a


Permanent Forum of People of African Descent to provide expert advice on addressing the challenges of
racism, racial discrimination, xenophobia and intolerance.

The resolution adopted by consensus by the [1]-member world body also calls for the forum to serve as
“a platform for improving the safety and quality of life and livelihoods of people of African descent” and
their full inclusion in the societies where they live.

The new body's creation comes ahead of the 20th anniversary of the controversial September [2] UN
World Conference Against Racism in Durban, South Africa, which was dominated by clashes over the
Middle East and the legacy of slavery. The US and Israel walked out during the meeting over a draft
resolution that singled out Israel for criticism and likened Zionism to racism.

That language was dropped in the final documents, which condemned and called for the eradication of
the scourges of racism, racial discrimination, xenophobia and intolerance.

48. Which of the following will replace [1] in the above passage?
(a) 189
(b) 190
(c) 193
(d) 196

49. UN declared the International Decade for People of African Descent as which of the following?
(a) 2011-2020
(b) 2014-2024
(c) 2015-2025
(d) 2021-2030

50. Which of the following is not true about Racism?


(a) Every year, 21st March, is observed as International Day for the Elimination of Racial Discrimination.
(b) Article 15, Article 16 and Article 29 of the Constitution of India prohibit discrimination on grounds of
“race”.
(c) India also ratified the International Convention on the Elimination of All Forms of Racial Discrimination
(ICERD) in 1968.
(d) Racial Discrimination is the specific geographical problem of Africa.

51. Which of the following will replace [2] in the above passage?
(a) 2001
(b) 2002
(c) 2003
(d) 2004

Head Office: 127, Zone II, MP Nagar, Bhopal |+91-7676564400| https://www.toprankers.com Page 13 of 40
52. Consider the following:
I. The forum will consist of 15 members
II. The resolution calls for the forum's first session to take place in 2022.
Which of the above is/are true?
(a) Only I
(b) Only II
(c) Both I and II
(d) Neither I nor II

Passage (Q.53-Q.57): Taliban insurgents tightened their grip on captured Afghan territory on Tuesday,
now controlling 65 per cent of the country, as US President Joe Biden urged the nation's leaders to fight
for their homeland. [1], capital of the northern province of Baghlan, fell to the Taliban on Tuesday
evening, according to residents who reported Afghan security forces retreating towards the Kelagi desert,
home to a large Afghan army base. [1] became the seventh regional capital to come under the control of
the militants in about a week. "Afghan leaders have to come together," Biden told reporters at the White
House, saying the Afghan troops outnumber the Taliban and must want to fight. "They've got to fight for
themselves, fight for their nation." The US president said he does not regret his decision to withdraw,
noting that Washington has spent more than $1 trillion over 20 years and lost thousands of troops. He
said the United States continues to provide significant air support, food, equipment and salaries to
Afghan forces. In Kabul, Afghan President Ashraf Ghani said he was seeking help from regional militias
he has squabbled with for years. He appealed to civilians to defend Afghanistan's "democratic fabric".

53. Which of the following replaces [1] in the above passage?


(a) Pul-e-Khumri
(b) Kunduz
(c) Kabul
(d) Herat

54. US-Taliban Agreement for withdrawal of American troops from Afghanistan took place in which of the
following?
(a) Kabul, Afghanistan
(b) Karachi, Pakistan
(c) Doha, Qatar
(d) Abu Dhabi, UAE

55. Which of the following dams in Afghanistan is known as the ‘Afghan-India Friendship Dam’?
(a) Kajaki Dam
(b) Grishk Dam
(c) Salma Dam
(d) Dahla Dam

56. Consider the following statements:


I. Taliban forces have gained control of over 65% of Afghanistan.
II. The Taliban recently assassinated Dawa Khan Menapal, Head of the Afghan government's media
information centre in Kabul city.
Which of the above statements is/are true?
(a) Only I
(b) Only II
(c) Both I and II
(d) Neither I nor II

Head Office: 127, Zone II, MP Nagar, Bhopal |+91-7676564400| https://www.toprankers.com Page 14 of 40
57. India proposed Afghanistan’s membership in the South Asian Association for Regional
Cooperation (SAARC) in which of the following years?
(a) 2001
(b) 2004
(c) 2005
(d) 2002

Passage (Q.58-Q.62): The closing ceremony in Olympic Stadium was fairly relaxed, and perhaps most
poignantly, it aimed to show the athletes a small taste of ordinary life in Japan --something they haven't
been exposed to due to pandemic restrictions.
It wrapped up more than two weeks of athletic competition and the largest international gathering to take
place during the pandemic. The ceremony celebrated the athletes, the volunteers and the organizers of
the postponed Tokyo Games, which involved about 230,000 people, including more than 41,000 people
who traveled from abroad.
"You inspired us with this unifying power of sport. This was even more remarkable given the many
challenges you had to face because of the pandemic," [1], the president of the International Olympic
Committee, told the athletes. "You give the world the most precious of gifts – hope."
And the Japanese organizers have now passed the torch to the next city hosting a Summer Games –[2].
The ceremony kicked off with a video showing some highlights from the events of the Games, and
fireworks lit up the sky. The scenes — across countries and sports — celebrated the efforts of all the
athletes, not medals in particular.
Then, flag bearers from each country walked in together, in a parade of colorful flags. The U.S. flag
bearer is javelin thrower Kara Winger who was selected by fellow athletes. Japan's flag bearer was
karate gold medalist Ryo Kiyuna.

58. Which of the following replaces [1] in the above passage?


(a) Thomas Bach
(b) Jacques Rogge
(c) Seiko Hashimoto
(d) Yoshiro Mori

59. What is the total number of medals won by India in the Tokyo Olympics 2020?
(a) 9
(b) 6
(c) 7
(d) 8

60. Who is the first Indian individual to win a Gold medal in the Olympic Games history?
(a) Neeraj Chopra
(b) Abhinav Bindra
(c) Manpreet Singh
(d) Milkha Singh

61. Which of the following replaces [2] in the passage?


(a) Brisbane
(b) Paris
(c) Sydney
(d) Shanghai

62. Who among the following was the flag-bearer for India in the closing ceremony of the Tokyo Olympic
Games 2020?
(a) Neeraj Chopra (b) Meerabi Chanu (c) Bajrang Punia (d) Susheel Kumar

Head Office: 127, Zone II, MP Nagar, Bhopal |+91-7676564400| https://www.toprankers.com Page 15 of 40
Passage (Q.63-Q.67): China launched its long-awaited emissions trading system on Friday, a key tool in
its quest to drive down climate change-causing greenhouse gases and go carbon neutral by [1]. The
scheme was launched with China, the world's biggest carbon emitter, seeking to take a global leadership
role on the climate crisis in the lead up to a crucial UN summit in November.
China has hailed it as laying the foundations for what would become the world's biggest carbon trading
market, forcing thousands of Chinese companies to cut their pollution or face deep economic hits.
The programme was launched just days after the [2] unveiled its detailed plan to achieve carbon
neutrality by 2050.
However deep questions remain over the limited scale and effectiveness of China's emission trading
scheme, including the low price placed on pollution.
More broadly, analysts and experts say much more needs to be done if China is to meet its
environmental targets, which includes reaching peak emissions by 2030.
China's economic and energy policies are becoming more aligned with the government's environment
goals, according to Zhang Jianyu, vice-president of Environmental Defense Fund China.

63. Which of the following replaces [1] in the passage?


(a) 2050
(b) 2055
(c) 2060
(d) 2065

64. What is the rank of India in the Global Climate Risk Index 2021?
(a) 5th
(b) 7th
(c) 9th
(d) 11th

65. Which of the following countries has zero carbon footprint as of 2021?
(a) Serbia
(b) Suriname
(c) Finland
(d) Denmark

66. Which of the following replaces [2] in the passage?


(a) Korea
(b) European Union
(c) United States of America
(d) Japan

67. Consider the following:


I. India is positioned as the third largest greenhouse gas emitter but also with among the lowest per
capita emissions
II. India’s Paris agreement target is to reach 40% non-fossils by 2030.
Which of the above is/are true?
(a) Only I
(b) Only II
(c) Both I and II
(d) Neither I nor II

Head Office: 127, Zone II, MP Nagar, Bhopal |+91-7676564400| https://www.toprankers.com Page 16 of 40
SECTION – C: LEGAL REASONING

Directions (Q.68 – Q.104): Read the comprehensions carefully and answer the questions based on it.

Passage(Q.68-Q.72): The Supreme Court of India in 1996 through the K.R. Lakshmanan judgment has
devised a guiding concept to govern the practice of online gaming/gambling. Games, where chance
predominates over skill, are forbidden, while games, where skill prevails over chance, are permitted. In
fact, in comparison to other online games, Courts have been rather demonstrative in establishing a space
for fantasy sports. Both the Punjab & Haryana and the Bombay High Courts have concluded that the
games offered by Dream 11 (Fantasy Cricket, Kabaddi, and Football, among others) demand skill,
knowledge, judgment, and attention and are thus not considered to be coming under the scope of online
gambling.
Wagering or betting involves the transaction of money or any type of property to guess the outcome of a
race, game, or any other unpredictable event. Section 30 of the Indian Contract Act makes agreements
by way of wager, void, and unenforceable. The essentials to wager constitute the performance of the
bargain depending upon the determination of an uncertain event where both the parties have mutual
chances of gain and loss and where neither party has control over the happening of the event one way or
the other.
Unlike games of chance, in E-sports the amount of skill and talent involved in mastering the game has led
to its professionalization. Also, in E-sports games, the prize money on winning is pre-declared and does
not depend on the number of people joining the contest as in the case of other online games.
[Extracted from 'Here’s how online gaming and E-sports are taxed in India' by Shivani Jha, published 14
June 2021 on The Financial Express

68. Virat created a new gaming platform where football enthusiasts from all over the world could come
together and make their own football teams by selecting players from different clubs. Players could bet on
the outcome of the match, which was dependent on whose team was objectively better. To make an
objectively better team, it was important to have a fundamental understanding of the strengths and
weaknesses of each member of the team. Does this game fall within the scope of online gambling?
(a) No, because only games where chance predominates over skill are forbidden and not games where
skill prevails over chance;
(b) No, because only games where skill prevails over chance are forbidden and not games where chance
predominates over skill;
(c) Yes, because only games where chance predominates over skill are forbidden and not games where
skill prevails over chance;
(d) Yes, because only games where skill prevails over chance are forbidden and not games where
chance predominates over skill.

69. X and Y entered into a contract. The contract stipulated that X would pay 10$ to Y if a head turns up after
every two tosses of a coin and, Y would pay 10$ to X if a tail turns up after every two tosses of a coin.
The coin was to be tossed by Shamsher Bagawat, who was an expert at tossing coins. Based on your
reading of the passage, is the contract between X and Y valid?
(a) Yes, because the performance of the contract depends on the determination of an uncontrollable and
uncertain event where both the parties have mutual chances of gain and loss;
(b) Yes, because the performance of the contract does not depend on the determination of an
uncontrollable and uncertain event as the coin was to be tossed by an expert;
(c) No, because the performance of the contract does not depend on the determination of an
uncontrollable and uncertain event as the coin was to be tossed by an expert;
(d) No, because the performance of the contract depends on the determination of an uncontrollable and
uncertain event where both the parties have mutual chances of gain and loss.

Head Office: 127, Zone II, MP Nagar, Bhopal |+91-7676564400| https://www.toprankers.com Page 17 of 40
70. Basanti and Gabbar enter into an agreement where Basanti promised to pay Gabbar a sum of Rs.
55,000 if India wins the world cup. India wins the match and there is an obligation on Basanti to pay the
amount to Gabbar but she fails to do so. Gabbar initiates a suit against Basanti for the recovery Rs.
55,000. Will Gabbar succeed in the recovery of the agreed amount?
(a) Yes, because the agreement is void and non-enforceable as it is a wagering agreement,
(b) No, because the agreement is based on the loss of one party at the gain of the other,
(c) Yes, because it is dependent on an uncertain event and both the parties have no mutual interest
except gain or loss,
(d) No, because the agreement is void and non-enforceable as it is a wagering agreement.

71. FancyBros Ltd launched a mobile application for a game called Chaar Patti. This game provides a
realistic experience of a casino with real professional dealers, and glamorous studios. Players can play
Chaar patti from the comfort of their homes and pay amounts for each round of cards securely through
Paytm. Many novice card players incurred huge debts on the application while playing Chaar Patti and
were unable to pay their debts. Can FancyBros Ltd recover this amount from these players?
(a) Yes, because playing Chaar Patti involves mutual chances of gain and loss;
(b) Yes, because Chaar Patti is a game of skill and does not constitute online gambling or wagering;
(c) No, because Chaar Patti is a game where skill prevails over chance and not chance predominating
over skill;
(d) None of the above

72. Anuradha launched a new company called Laxmi Ltd. The company organised a sweepstake competition
where customers were asked to purchase tickets. A lottery was pulled out and the tickets which
contained the number ‘53’ were awarded cash prizes. Raju had purchased 50 tickets in the hopes of
winning a cash prize and ended up winning Rs. 1 lakh in total. However, when he went to claim the
amount, he was turned away. Can he sue Laxmi Ltd for recovery of the amount?
(a) Yes, because a valid contract was constituted between Raju and Laxmi Ltd when he purchased 50
lottery tickets;
(b) Yes, because lotteries do not have the characteristics of wagers and are enforceable;
(c) No, because lotteries are fiscally irresponsible and discouraged by the government;
(d) None of the above.

Passage(Q.73-Q.77): In a recent ruling, the Supreme Court of India observed that the ‘doctrine of
impossibility’ would be equally applicable to Court orders.
The Doctrine of Impossibility stems from the Latin maxim, lex non cogit ad impossibilia, signifying that ‘a
man cannot be compelled to do what is impossible’. The genesis of the doctrine is rooted in the law of
contracts. Statutorily enshrined in Section 56 of the Indian Contract Act, 1872, this provision allows
contracts to be set aside due to supervening impossibility preventing its performance.
The application of the doctrine to court orders reflects a shift in the non-performance of a statutory
obligation to a prospective assumption of non-compliance with the Court’s directions by the Executive.
Courts often function within their boundaries to enforce directions for public welfare. Although some of
these directions may be stringent, they go a long way in ensuring that states wake up to the seriousness
of the matter concerned. Courts generally take into account the response of the Executive before issuing
directions to them. Thus, it would lead to a lack of clear demarcation between the mandatory directions
issued by the courts and the advice or opinions given in a general context, which would make way for the
Executive to excuse themselves from the directions of the court.
As one of the three pillars of democracy, courts are often approached to ensure that states fulfil their
responsibilities owed to the citizens. This, however, does not excuse the governments for failing their
duties on the grounds of impossibility and impracticality. Such directions cannot be circumvented by
claiming ‘impossibility’ or impracticality’ even if it appears to be difficult to implement.
[Extracted with revisions from 'Extending the Doctrine of Impossibility to Court Orders: A Conundrum' by
Aisiri Raj and Vishnu Mohan Naidu, published on NLUJ Criminal law blog

Head Office: 127, Zone II, MP Nagar, Bhopal |+91-7676564400| https://www.toprankers.com Page 18 of 40
73. During the peak of COVID-19 pandemic, the Allahabad High Court issued several directions such as
providing a minimum number of ambulances in graded towns, requiring new firms to manufacture
vaccines by borrowing the formula from existing manufacturers, providing oxygenated beds in all state
nursing homes and upgradation of state medical colleges within a very short period. The State
government contests that these directions are nearly impossible to implement and the directions
regarding the manufacture of vaccines could also have international ramifications. Can the doctrine of
impossibility be invoked against the directions of the Allahabad High Court?
(a) Yes, because the doctrine of impossibility can be invoked against Court orders when there is
supervening impossibility preventing their performance;
(b) Yes, because though directions are difficult to implement and impractical, they are technically
possible;
(c) No, because the government cannot be excused for failing their duties on the grounds of impossibility
and impracticality;
(d) No, because invoking the doctrine of impossibility will lead to a lack of clear demarcation between the
mandatory directions issued by the courts and the advice or opinions given in a general context.

74. In the case of River Ganga Organisation v State, the Court ruled in favour of the NGO, River Ganga
Organisation. The NGO had sought a clean-up of River Ganga, including the ghats and the
implementation of mandatory awareness drives, setting up of toilets etc. which was granted by the Court.
Three months later, the State said a lack of funds prevented them from controlling the rampant
environmental pollution and fulfilling the court order. Can the State invoke the doctrine of impossibility to
avoid the performance of its duties?
(a) Yes, because a lack of funds can lead to difficulties in the implementation of environmental policies;
(b) Yes, because a lack of funds constitutes a supervening impossibility which prevents performance of
the order;
(c) No, because there is a pre-existing duty on the government to address these problems by prioritising
them and allocating a budget to meet these requirements;
(d) No, because a mere difficulty in implementation due to lack of funds does not make the order
impossible.

75. Aladin and Jasmine enter into a contract. Aladin agrees to discover treasure by magic with the help of his
Magic Carpet and a genie. Jasmine had neither seen nor heard of a Magic Carpet or genie but, she
promises to give a Rolls Royce car to Aladin if he performs his part of the bargain. Jasmine grows
impatient and demands performance of the Contract. Based on the principles mentioned in the passage,
can she do so?
(a) No, because discovering treasure by magic with the help of a Magic Carpet and a genie is not
impossible;
(b) No, because Aladin cannot be compelled to do what is impossible;
(c) Yes, because Aladin can be compelled to do what is impossible;
(d) Yes, because discovering treasure by magic with the help of a Magic Carpet and a genie is not
impossible.

76. Manjeet booked a banquet hall for his daughter's wedding. The contract with the owner of the hall
stipulated that the owner would be liable to pay damages to Manjeet if he cancelled the booking without a
one-month notice. Two days before the wedding, an unforeseen short circuit culminated into a large fire
which burnt down the hall. Although he felt bad for the owner of the hall, Manjeet was angry because he
would not be able to book a venue on such short notice. Can he seek damages from the owner of the
hall?
(a) Yes, because a short circuit does not amount to a supervening impossibility as the owner should not
have been negligent;
(b) No, because a short circuit amounts to a supervening impossibility and renders the contract void;
(c) Yes, because a short circuit amounts to a supervening impossibility that can set aside the contract;
(d) No, because a short circuit amounts to a supervening impossibility that can set aside the contract.

Head Office: 127, Zone II, MP Nagar, Bhopal |+91-7676564400| https://www.toprankers.com Page 19 of 40
77. Y leased X’s entire restaurant at a rate higher than usual. Y agreed to pay a higher price expecting huge
profits as the Olympics were going on and a large number of tourists were expected in the city.
Subsequently, the government passed an order banning all tourists in the country due to the rising
number of COVID-19 cases. Consequently, no profit was materialised for Y as expected. Y pleaded
impossibility because he did not want to pay the higher rent any longer. Will Y’s plea be accepted?
(a) Yes, because it is impossible for him to attain huge profits in light of the government order and that
was the entire basis of the contract;
(b) Yes, because it is impossible for Y to perform his end of the bargain as he would suffer unprofitability;
(c) No, because mere unprofitability cannot render the contract to be frustrated;
(d) No, because the terms of the contract are not impossible to perform for Y and they are only
unprofitable.

Passage(Q.78-Q.82): The proviso to Section 43D(5) of Unlawful Activities (Prevention) Act [UAPA] lays
down that a court shall not allow bail if “there are reasonable grounds for believing that the accusation
against such person is prima facie true,” thus leading to the belief that courts have no power to grant bail
to a person accused under Sections 15 to 23.
Section 13 mainly concerns itself with disrupting Sovereignty, whereas Section 15 makes punishable, any
terrorist act and opens with the words “any act with intent to threaten or likely to threaten the unity,
integrity, sovereignty of India, or with intent to strike terror or likely to strike terror in the people..." by
using lethal weapons, bombs etc.
The conclusion that the court is not to presume the guilt of the accused, and that the "burden to
demonstrate the prima facie veracity, the allegation must fall upon the prosecution" is absolutely correct.
When UAPA uses the words “prima facie true”, it meant that the court must accept the guiltof the accused
persons, even if on broad probabilities. This could happen only if the prosecution had discharged its initial
duty of establishing a prima facie case.
Why this reasoning is sound can be tested by the following simple explanation. At the stage when the
court is considering bail, the only material available with the court is that which has been brought or
collected by the prosecution. So evidently, the court can make up its mind on that material alone which
needless to say should be sufficient for it to arrive at an opinion that prima facie the case is true.
[Extracted with revisions from ‘Delhi Riots case: How the Delhi High Court caught the bull by the horns
while confronting Section 43(D)5 of UAPA' by Anjana Prakash, published 17 June 2021 on bar and
bench

78. Sharjeel participated in a protest against the established, democratically elected Government of India.
The protest gained widespread media coverage and large masses gathered in the concerned ground.
Eventually, the crowd became so rowdy that police was ordered to do a lathi charge to clear the space.
Sharjeel was charged under Section 15 of UAPA. When his case came for hearing, the state submitted
pictures of Sharjeel addressing the crowd along with videos of his speeches. Has the state discharged its
burden under Section 43(D)5 of UAPA?
(a) Yes, because the state has submitted enough material on record for the Court to make up its mind
and decide if the accusations are prima facie true;
(b) No, because the state has submitted enough material on record for the Court to make up its mind and
decide if the accusations are prima facie true;
(c) Yes, because the state has not submitted enough material on record for the Court to make up its
mind and decide if the accusations are prima facie true;
(d) No, because the state has not submitted enough material on record for the Court to make up its mind
and decide if the accusations are prima facie true.

Head Office: 127, Zone II, MP Nagar, Bhopal |+91-7676564400| https://www.toprankers.com Page 20 of 40
79. Based on the evidence presented by the State against Sharjeel, as enumerated in the previous question,
the Court denied Sharjeel’s application of bail. The Court found the allegations against him to be prima
facie true. Sharjeel objected to this decision because he believes that he has a right to bail. Is the
disposal of Sharjeel’s bail application by the Court, a valid action?
(a) Yes, because jail is the exception and bail is the rule in criminal law;
(b) No, because the Court has the power to grant bail in UAPA cases even if the allegations are prima
facie true;
(c) Yes, because the Court only needs to make up its mind that the allegations are prima facie true to
disallow bail application;
(d) No, because there is not enough material for the Court to determine if the allegations against Sharjeel
are prima facie true.

80. Tabrez was arrested on charges of rioting by the police of Koregaon for threatening the sovereignty,
integrity and unity of India, under Section 15 of UAPA. Besides the witness statements of a few people
who were not even on the spot of the alleged rioting, the police did not submit any other evidence to
substantiate the charges. Can the Court grant bail to Tabrez in this case?
(a) Yes, because there are reasonable grounds for believing that the accusations against Tabrez are
true;
(b) Yes, because the police did not discharge its burden of proving that the allegations against Tabrez
are prima facie true;
(c) Yes, because the police discharged its burden of proving that the allegations against Tabrez are
prima facie true;
(d) No, because there are no reasonable grounds for believing that the accusations against Tabrez are
true.

81. P and Q prepared petrol bombs with empty glass bottles of Corona beer. They were loyal citizens and
were happy with the government’s functioning. However, they were very bored with their miserable,
monotonous lives and wanted to get a ‘kick’ out of doing something different. As a result, they decided to
throw the petrol bombs in a public park at 2 am in the night when there were hardly any people in the
park, so as to create a scare. Can they be convicted under the provisions of UAPA?
(a) Yes, because they disrupted sovereignty of the nation under Section 13 of UAPA;
(b) Yes, because they intended to strike terror in the people of the nation under Section 15 of UAPA;
(c) Yes, because they committed an act with the intention to threaten the unity and integrity of the nation
under Section 15 of UAPA;
(d) None of the above.

82. P and Q’s antics were published in the newspapers and they were identified as the culprits behind the
petrol bomb incident after a swift police investigation. The Court was informed that materials used in
making the petrol bombs, similar to the ones that were thrown in the park, were found at P and Q’s
house. Can the Court grant bail to P and Q?
(a) Yes, because there are reasonable grounds for believing the accusations against P and Q are prima
facie true;
(b) No, because there are no reasonable grounds for believing the accusations against P and Q are
prima facie true;
(c) No, because there are reasonable grounds for believing the accusations against P and Q are prima
facie true on broad probabilities;
(d) No, because there are no reasonable grounds for believing the accusations against P and Q are
prima facie true even on broad probabilities.

Head Office: 127, Zone II, MP Nagar, Bhopal |+91-7676564400| https://www.toprankers.com Page 21 of 40
Passage(Q.83-Q.88): The offense of criminal breach of trust, as defined under section 405 of IPC, is
similar to the offense of ‘embezzlement’ under the English law. A reading of the section suggests that the
gist of the offense of criminal breach of trust is ‘dishonest misappropriation’ or ‘conversion to own use’
another’s property, which is nothing but the offense of criminal misappropriation defined u/s 403.
The essential ingredients of the offense of criminal breach of trust are (1) The accused must be entrusted
with the property or with dominion over it, (2) The person so entrusted must use that property, or; (3) The
accused must dishonestly use or dispose of that property or wilfully suffer any other person to do so in
violation, (a) of any direction of law prescribing the mode in which such trust is to be discharged, or; (b) of
any legal contract made touching the discharge of such trust.
For criminal breach of trust, the accused is entrusted with property or with dominion or control over the
property. As the title to the offense itself suggests, entrustment ofproperty is an essential requirement
before any offense under this section takes place. The language of the section is very wide. The words
used are ‘in any manner entrusted with property’. So, it extends to entrustments of all kinds-whether to
clerks, servants, business partners or other persons, provided they are holding a position of trust. The
term “entrusted” found in Section 405, IPC governs not only the words “with the property” immediately
following it but also the words “or with any dominion over the property.”
There are two distinct parts involved in the commission of the offense of criminal breach of trust. The first
consists of the creation of an obligation in relation to the property over which dominion or control is
acquired by the accused. The second is misappropriation or dealing with the property dishonestly and
contrary to the terms of the obligation created. The principal ingredients of Criminal Breach of Trust are
thus ‘entrustment’ and ‘dishonest misappropriation’.

83. The Crime Department is investigating a major bank fraud case. One of the employees X, who is
entrusted with handling the matter has lost an important file which has information on the whereabouts of
the accused. Decide.
(a) X has committed criminal breach of trust as he dishonestly disposed of the property;
(b) X has not committed criminal breach of trust as he was not dishonest in disposing of the property;
(c) X has committed criminal breach for he was bribed;
(d) X’s liability cannot be determined.

84. Please refer to the facts above. An intel is received that there was a phone call between the accused and
X before the file went in missing. Decide.
(a) X has committed criminal breach of trust as he dishonestly disposed of the property;
(b) X has not committed criminal breach of trust as he was not dishonest in disposing of the property;
(c) X has committed criminal breach for he was bribed;
(d) X’s liability cannot be determined.

85. Anil and Bhadu are working at the police department. A phone call was traced at Bhadu’s place late at
night. In addition to a phone call, a man wearing plain clothes was also seen taking money to Bhadu’s
place and documents concerning an important case went missing on the next day. This is viewed as
credible evidence of bribery. Decide.
(a) Bhadu has committed criminal breach of trust as he dishonestly disposed of the property;
(b) Bhadu has not committed criminal breach of trust as he was not dishonest in disposing of the
property;
(c) The very allegation of bribery is enough to show that Bhadu has committed criminal breach of trust;
(d) Bhadu is a public servant within the meaning of the IPC and thus, he cannot be held liable for criminal
breach of trust.

Head Office: 127, Zone II, MP Nagar, Bhopal |+91-7676564400| https://www.toprankers.com Page 22 of 40
86. Albert wanted to give some documents to his junior. The documents were entrusted to the junior X over a
phone call. X agreed. After hanging up the phone, Albert asked the peon to deliver the documents to X’s
office. The file never reached X’s office.
(a) The peon is liable for criminal breach of trust as he was entrusted with the documents.
(b) The peon is not liable for criminal breach of trust as he was not entrusted with the documents. He
only delivered it.
(c) The peon is liable for criminal breach of trust and X is vicariously liable.
(d) The peon is liable for criminal breach of trust and Albert is vicariously liable for he was the one who
entrusted the file to the peon.

87. Please refer to the facts above. It turns out that the peon had spilt some water onto the file. Scared, he
put the file out in the sun and lied to X about the superior not giving the file. Decide.
(a) The peon is liable for criminal breach of trust as he misappropriated the property with a dishonest
intention.
(b) The peon is not liable for criminal breach of trust as he did not have a dishonest intention.
(c) The peon is liable for criminal breach of trust as he lied to X about his superior and thus has a
dishonest intent.
(d) The mere fact that the peon lied about it, is enough to show the presence of dishonest intention.

88. Anil took a file along with him with the intent of taking it away from the police station. Till this time, there is
no evidence of Anil accepting a bribe.
(a) Anil has committed criminal breach of trust as he dishonestly disposed off the property;
(b) Anil has not committed criminal breach of trust as he was not dishonest in disposing of the property;
(c) Anil has not committed criminal breach of trust for he has not misused the property;
(d) Anil’s liability is subject to further evidence of causing the government wrongful loss which is the
accepted metric of a dishonest intention.

Passage(Q.89-Q.94): The offence of criminal conspiracy is defined under Section 120-A of Chapter V-A
of the Indian Penal Code, 1860. According to Section 120-A, when two or more persons come together
and agree to do, or cause something to be done, which constitutes an illegal act or a legal act carried
forward by illegal means, such persons would be guilty of the commission of the offence of criminal
conspiracy. In simpler terms, conspiracy refers to the meeting of minds for the commission of an offence.
However, no such agreement would constitute the offence of criminal conspiracy, unless and until an act
is performed in furtherance of such an agreement. The explanation attached to Section 120-A makes it
clear that it is immaterial whether the illegal act committed in furtherance of such an agreement, is the
focal point of the agreement or, is merely incidental to the performance of the ultimate goal of the
agreement.
Salient features: There must be two or more person involved in the commission of the offence of criminal
conspiracy; there must be an agreement between the parties; such an agreement must be for the
commission of an illegal act or the commission of a legal act by illegal means; When the agreement is for
the commission of a legal act by illegal means, an overt act must have been carried out by the parties in
furtherance of the same, and the mere agreement is not sufficient to establish the commission of the
crime.
For the establishment of the offence of criminal conspiracy, the existence of an agreement between the
parties is a sine qua non. This agreement may be express or implied, the important factor is consensus
ad idem, i.e., meeting of minds. The agreement must be read as a whole and the object be ascertained.
It is not necessary that more than one person must always be convicted for the offence of criminal
conspiracy, it is sufficient if the court is convinced that more two or persons were actually involved in the
conspiracy. The offender might join the conspiracy from the start itself or at any time before the
completion of the objective of the agreement, irrespective of the time of joining, each party to the offence
would be held equally responsible. A criminal conspiracy is said to persist as long as the parties to the
agreement continue to act in furtherance of the objects of the agreement.

Head Office: 127, Zone II, MP Nagar, Bhopal |+91-7676564400| https://www.toprankers.com Page 23 of 40
89. A and B decided to rob a house of Mr. D. In the process of doing so, A killed Mr. D’s son. Is there a
criminal conspiracy under Section 120A?
(a) Yes, both A and B are part of the criminal conspiracy.
(b) No, only A is responsible for the criminal conspiracy as there was no agreement between A and B for
the murder but only for the robbery.
(c) There is no criminal conspiracy as the essential ingredients for such an act are prior agreement and
involvement of 2 or more people. .
(d) B has not committed criminal conspiracy.

90. In the above facts, if they blackmailed Mr. D’s guard to help them in the robbery, is it a criminal
conspiracy?
(a) Yes, all three of them are part of the criminal conspiracy under Section 120A.
(b) Only A and B form part of the criminal conspiracy.
(c) This is a case of robbery and not criminal conspiracy.
(d) There is no criminal conspiracy since there was no prior agreement between the guard and A & B.

91. On Monday, Ashfaq and Bittu committed theft in a house. In addition, there were three other persons
Paras, Qasim and Rohit who happened to rob the same house on the same day. All five of them were
caught by the police. Is there a commission of criminal conspiracy?
(a) All five of them formed the part of criminal conspiracy.
(b) Only Ashfaq and Bittu formed the part of criminal conspiracy.
(c) Only Paras, Qasim and Rohit formed the part of criminal conspiracy.
(d) There is no criminal conspiracy.

92. In the above facts, Qasim wanted to kill the house owner’s son due to his old rivalry with him but Paras
and Rohit did not agree to this as they only wanted to rob the house. Qasim killed the owner’s son after
taking their all money and jewellery.
(a) All three of them committed criminal conspiracy.
(b) Only Qasim committed criminal conspiracy.
(c) There is no criminal conspiracy as the prior agreement was only of robbing the house and not murder
of the owner’s son. Therefore, Paras and Rohit are not responsible for the murder committed by
Qasim as they had intention to do so.
(d) This is a case of murder and not criminal conspiracy.

93. Paritosh runs a factory that manufactures incense sticks. A few of his workers let in Wazir and Raman
who robbed the factory.
(a) All of them committed criminal conspiracy as there is meeting of minds in the commission of the
crime.
(b) Only Wazir and Raman are responsible for criminal conspiracy.
(c) This is a case of robbery and not criminal conspiracy.
(d) No criminal conspiracy is committed.

94. Mr. Bharath is a student of B.E. in Computer Science He loves his computer very much. He considers his
computer as his close friend and companion. On 1.4.2006, while interacting with his computer, he hacked
into the Bank account of Mr. Javed and was successful in withdrawing money front Mr. Javed’s bank
account. He did it to please his girlfriend.
(a) Mr. Bharath has committed an offence.
(b) Mr Bharath has committed cyber crime.
(c) Mr Bharath has committed offence of criminal conspiracy.
(d) Mr.Bharath has not committed the offence of criminal conspiracy.

Head Office: 127, Zone II, MP Nagar, Bhopal |+91-7676564400| https://www.toprankers.com Page 24 of 40
Passage(Q.95-Q.100): A journalist is not expected to dramatise an incident and place the subject of the
news report at risk; the Allahabad High Court observed while hearing a case where a reporter allegedly
egged on a person to die by suicide in front of the Uttar Pradesh assembly, promising him airtime.
The deceased person's wife is the complainant in this case.
A journalist is not expected to dramatise a sensational and horrifying incident and make news by putting
his actor in a pitiable condition in danger of death.
The bench also spoke on the role of a journalist "The journalist keeps an eye on the anticipated or
sudden events happening in the society and brings them to the information of all the people through
various news media without any tampering; this is his business."
The role of journalists in exacerbating or propagating a situation thrust into public view was also critiqued
in the aftermath of the passing of actor Sushant Singh Rajput last year, when several mainstream
television news channels put Bollywood actors on a media trial and conducted televised investigations on
their own, going to the extent of pronouncing selected people guilty.
If you know someone – a friend or family member – at risk of suicide, please reach out to them. The
Suicide Prevention India Foundation maintains a list of telephone numbers they can call to speak in
confidence. Icall, a counselling service run by TISS, has maintained a crowd sourced list of therapists
across the country. You could also take them to the nearest hospital.

95. Magan is a very prosperous businessman in the community of Hirania Nagar. One fine day, the news
issues an alert of a possible bombing in the area due to an escaped terrorist named Mangal. However,
the news fails to display his name correctly and instead of 'Mangal', displays 'Magan', which leads to a lot
of commotion in Magan's vicinity as he was well known throughout the town. This led to him being
insulted and disgraced in his community, which led to him getting depressed and committing suicide. The
Media thereafter issued an apology when the state of affairs came to its notice. Would the media be
liable for his actions herein?
(a) Yes, as his name was severely besmirched, which led to his suicide.
(b) No, as the community knew that he was not the bomber.
(c) Yes, as he was utterly disgraced, and his reputation was destroyed.
(d) No, as any sane person would not equate a well-known businessman with a terrorist bomber.

96. In the above set of facts, had Magan not committed suicide, would the media house have still been liable
for his state of affairs?
(a) No, as he did not commit suicide or harm himself in any other way and as subsequently apologised
to.
(b) Yes, due to the false information being published by them, the media house should be held liable.
(c) No, as after issuing an apology, everybody would have realised the mistake.
(d) None of the above.

97. Aaj Tak Sarva Satark is a very well-known news station in North India, with almost all of the households
in that area relying on its bulletins. Recently, they have ventured into the South Indian region with a view
to capturing the populace therein. Consequently, when they receive a hot tip about one big gangster
living on the outskirts of Kerala, they immediately decide to report it. To their dismay, the person living
there was a poor fisherman trying to make ends meet, and someone had pranked them by sending his
particulars. This leads to his house being flooded by police authorities but to no avail, and the fisherman
subsequently goes into a shock caused by the entire trauma. Is the news station liable to be held guilty
for exacerbating and dramatising?
(a) Yes, the news station should be held liable as their reputation led the masses to believe that the
person living there was a terrorist.
(b) No, as the news station was a relatively new one in this vicinity and had not yet garnered any support
yet.
(c) Yes, as its support is irrelevant here, they reported false news.
(d) No, as the person living there was not a celebrity, and neither did they pose a danger to his life.

Head Office: 127, Zone II, MP Nagar, Bhopal |+91-7676564400| https://www.toprankers.com Page 25 of 40
98. The same news channel as above reported the relationship of Dhanveer Kapoor and
BeepshikaPadukone as being an abusive and toxic one, wherein both the people involved preach
violence. The people involved, however, did not make any public statement about their relationship and
were only ever seen as having dinner once in Mumbai. However, after this news telecast, they received a
lot of backlash from the public, which led to them being pelted with eggs and their movies flopping. Would
this action of the Media make them liable for the danger to the actors' life?
(a) Yes, as their news roused the public, which led to their movies flopping and them being assaulted.
(b) No, as their relationship was public knowledge anyway.
(c) Yes, as their statements led to the whirlwind of events in which they were caught.
(d) No, as their statements did not endanger their lives.

99. The same news channel as above receives a request by Dhananjaya Bhat; a celeb turned politician who
had allegedly helped a few terrorists enter India through his private cruise. The celeb now wants to
confess about the same. He contacts the news channel stating his request and is granted the same.
However, while on primetime, his phone and anxiety medication is taken by the crew so that the show
isn't interrupted. However, this leads to him having an anxiety attack, and when that gets out of control,
he suffers from a stroke which leads to his demise. Would this fiasco lead to the news station being
implicated in the death of the celeb?
(a) Yes, as the actions of the news station led to his ultimate demise.
(b) No, as the news station crew did not force him to surrender his medication.
(c) Yes, as the news channel crew failed to accommodate his health condition.
(d) None of the above.

100. In the above set of facts, after his interview on the news channel, Dhananjaya gets home but is
immediately surrounded by people all around the house and is pelted with stones and fire-lit bottles. This
leads to him getting lynched eventually and the public rioting against his family as being a threat to the
sovereignty of the country. The media learns about this and immediately tries reaching the parents of the
celeb and continue to do so even after being turned down. Would the media be liable here?
(a) The media would be liable here both for the death of the celeb and troubling his parents.
(b) The media would only be liable for troubling his parents.
(c) The media would only be liable for the death of Dhananjaya.
(d) The media will not be liable for either of the events.

Passage(Q.101-Q.104): The Delhi High Court said that a safe house has to be provided for all types of
couples- inter-faith, inter-caste, those belonging to the LGBTQIA+ community - facing opposition from
families. While hearing a petition filed by a couple in a similar situation, Justice Mukta Gupta said.”
Whether it’s inter-caste, inter-religion, inter-community….LGBTQ also covered”. “SHO Police Station
Mayur Vihar Phase 1 to ensure petitioners taken from office and lodged safely at the safe house….there
they would be given necessary protection from SHO through beat staff and other security staff….” The
Court ordered. The Court pointed out that the current arrangement was only a “stop-gap” measure and
that the couple’s families had to be made parties to the case in the future. “Someday they will have to
settle on their own. It’s only a stop-gap….They can live at the safe house for some time but eventually
have to move out….You need to implead the family members….”, Justice Gupta said. Appearing for the
petitioners, Advocate Utkarsh Singh submitted that his clients were adults but their families assaulted
them. The petitioners, one of whom was NGO Dhanak For Humanity, argued that the couple had
approached the DCP concerned, but weren’t provided with a safe house as yet. The petitioner also
feared being tracked by the families and the people known to them. Judicial intervention was sought by
the couple on the ground that they also “have the right to pursue their lives with dignity irrespective of
their sexual orientation”. Not providing them with the safe house was violation of their fundamental right
guaranteed under Article 21 of the Constitution of India, it was argued.
Source name: Bar and Bench

Head Office: 127, Zone II, MP Nagar, Bhopal |+91-7676564400| https://www.toprankers.com Page 26 of 40
101. A and B are couple aged 19 and 17, residing in More Vihaar are in the state of Dailhi. They were
threatened by B’s family that if she doesn’t leave A, they will create trouble for both of them. The couple
took the help of the state police but the police counselled them and told them to return to their respective
homes and try to convince their parents. In the light of the passage given above, select the most
appropriate option.
(a) State police is justified in its action because, the girl is a minor and she must not live with her lover in
a live-in relationship.
(b) The girl is a minor and at this tender age, children are not able to take the right decision of their own,
so the state police authority is justified in its action.
(c) Although the girl is a minor, but having A that is her male partner a major, so police should give a
helping hand to the couple from their families.
(d) Although, the girl is a minor and the male partner is a major and they own a right to get protection
from the police authority regarding the matter.

102. P and Q were boys who were in love with each other and called themselves a couple and never hesitated
to express their love in mild way anywhere, even in public. After disclosing their truth to their families,
they never objected their relationship but had a problem of their open expression of the same. After
repeated reminder, they didn’t stop doing the same and ultimately families of the couple ordered them to
get separated, negating which the state police gave them shelter in the safe house. But frequent
expression of their love agitated the police and the authority ordered them to leave the place with
immediate effect. In the light of the paragraph, opt out the most appropriate option.
(a) Providing safe house to a couple resorting to obscenity is nowhere justified and thus, the police have
done right.
(b) Every couple has the right to express love anywhere to any extent because now the same sex
relation or marriage has been legalized by the Hon’ble Supreme Court of India.
(c) Couple must be given the allowance to take shelter in the safe house because mild expression of
love is couple’s right and there is no harm in it.
(d) Couple must be given to take shelter in the safe house because they have been threatened by their
families if not got separated from each other.

103. B and C, both aged 17 were couple who took shelter in the safe house and were living there from 3
months escaping from their family members and their acquaintances who were against their relation.
After repeated counselling and certain advices of the caretaking police authority, they were adamant not
to confront their families in any form. The police resorted to strict action and forced them to leave the safe
house and confront their families and convince them about the same. In the light of the above passage,
opt out the most appropriate option.
(a) Actions of the police of forcing the couple to leave the safe was nowhere justified as taking the shelter
in the safe house is their right and giving protection to the couples is police’s duty.
(b) Police’s action was somewhere justified because a couple can’t live like this whole life, and someday,
they have to face the fact and confront their families to convince them. Thus the actions of the police
were justified.
(c) Police owns the right and duty to take a decision for the couple for the betterment of the couple as
they were both minors and it is the duty of the police to make them realize their mistakes to ensure a
civil society.
(d) Forcing a couple to do anything is nowhere justified and as, the couples were threatened by their
family members, it was the duty of the police to give them protection.

Head Office: 127, Zone II, MP Nagar, Bhopal |+91-7676564400| https://www.toprankers.com Page 27 of 40
104. From the above passage, which of the option gives a perfect gist of the passage?
(a) Couples have the right to be protected by the police in the safe house irrespective of their explicit
expression of love anywhere to any extent.
(b) Safe house is meant for providing only a temporary shelter to the couples.
(c) Safe house is meant for all the couples irrespective of their sexual orientation but not for minors living
in a relationship.
(d) Police has the authority over the minor to force them out of the safe house to confront their families
regarding their relationship because minors are generally not capable of taking their lives’ decision
decently.

Head Office: 127, Zone II, MP Nagar, Bhopal |+91-7676564400| https://www.toprankers.com Page 28 of 40
SECTION - D: LOGICAL REASONING

Passage(Q.105-Q.109): One hundred years of the CCP is an occasion for the Chinese to celebrate.
Most importantly, disproving Western negativity, the phenomenal success, the unprecedented growth
rate and eliminating poverty for 800 million people are reasons worthy of celebrations. It is not correct to
say that the CCP has not accepted past mistakes and taken the necessary corrective measures. These
have been the bases on which Deng Xiaoping initiated reforms that made possible the economic miracle
that followed. The Chinese model that Xi applauds is based on an impressive track record and its
legitimacy is not limited to the renewal of the popular mandate as we are used to in liberal democracies,
but in projecting a political structure that is free from compulsions of electoral politics.

The entire effort of the CCP is to remind China and the world of the century of humiliation that it suffered
because of the Western design to perpetuate China’s semi-colonial and semi-feudal order. The century
that began with the Opium Wars denied China its rightful evolution and the supremacy that it enjoyed as
the longest civilizational state in the world. Restoring China to its rightful place is not an ideological
question but a nationalistic one, which the CCP intends to achieve by the acquisition of technology and
achieving rough parity by facilitating frontier technology. China is aware of the wide differences between
the Cold War periods and accepts the fact of an interdependent world. The BRI project is an example of
that interdependence. The important point for China is the full restoration of its sovereignty in which all
other considerations become secondary.

Nationalism being the driving force, the CCP accepts mistakes and setbacks, but emphasises the
economic and cultural aspects that have made it secure and powerful. This reference by Xi not to accept
any bullying is with regard to the unequal treaties that the Western powers and Japan inflicted on China
in the past. The whole emphasis is about restoring China to its pre-1840 status. The point is that China’s
evolution is very different from the West and the CCP’s legitimacy is based on the centrality of its
capacity to guarantee China’s independence. China reminds the West of its duplicity as well: When the
latter invested so heavily, singularly contributing to China’s success, it did so without insisting on Beijing’s
track record on human rights and democratic values. But these have now become of paramount
importance with the rise of China as a formidable power and rival.
[Excerpt from an article by Subrata Mukherjee, The Indian Express, July 5, 2021:

105. All of the following can be inferred except?


(a) Western countries are also responsible for the economic growth of China.
(b) China’s influence internationally has increased substantially in the last few years.
(c) China has become the superpower of the world.
(d) China knows the present world order and takes steps keeping that in mind.

106. Which of the following conclusions can be reasonably drawn from the passage above?
(a) Nationalism is one of the major force behind legitimisation of CCP’s authority in China.
(b) West has become weaker as compared to China in the present world order.
(c) Xi is the most powerful leader China ever had.
(d) None of the above

107. Which of the following weakens the author’s arguments?


(a) China is trying to increase its influence on other countries of the world.
(b) Even after the phenomenal success and unprecedented growth rate of China, poverty in China still
exist.
(c) The threat of Chinese acquisition of Taiwan still exist.
(d) None of the above

Head Office: 127, Zone II, MP Nagar, Bhopal |+91-7676564400| https://www.toprankers.com Page 29 of 40
108. Which of the following is an assumption the author makes?
(a) Human rights and democratic values infringement still exist in China.
(b) CCP has been reasonably successful in strengthening economic prosperity of China.
(c) Both A) and B)
(d) None of the above

109. If author’s arguments in the passage above are true, which of the following is most likely to be true?
(a) West has always been and always will be most powerful in the world.
(b) West has started seeing a more powerful China as its rival.
(c) Western countries are more powerful as compared to China.
(d) All of the above

Passage(Q.110-Q.113): There’s a new virus in town and it’s not fooling around. You can catch it through
face-to-face contact or digitally – that is, via a human or a bot, thanks to technology. Few of us possess
immunity, some are even willing hosts; and, despite all we’ve learned about it, this virus is proving more
cunning and harder to eradicate than anyone could have expected.

Misinformation isn’t new, of course. Fake news was around even before the invention of the printing
press, although the first large-scale journalistic sham occurred in 1835, when the New York Sun
published six articles announcing the discovery of life on the Moon (specifically, unicorns and bipedal
beavers). Consider, too, early modern witch hunts, or those colonial myths that depicted slaves as a
different species; the back-and-forth volleys of anti-Jewish and anti-German propaganda during the world
wars, McCarthyism’s Red Scare, even communism’s utopian narratives. History teems with deceit.

What’s different today is the speed, scope and scale of misinformation, enabled by technology. Online
media has given voice to previously marginalized groups. The transmission of falsehoods now spans a
viral cycle in which AI, professional trolls and our own content-sharing activities help to proliferate and
amplify misleading claims. Moreover, these new developments have come on the heels of rising
inequality, falling civic engagement and fraying social cohesion – trends that render us more susceptible
to demagoguery. Just as alarming, a growing body of research over the past decade is casting doubt on
our ability – even our willingness – to resist misinformation in the face of corrective evidence.
Extracted with edits from Aeon.co:

110. Which among the following best describes the author’s arguments?
(a) The author believes that technology has brought with itself more misinformation to previously
marginalized groups than ever before.
(b) The author talks about the scale of misinformation in the world of this century and ways to mitigate its
negative effects.
(c) The author explains that misinformation has been around for a long time but there is a difference
between misinformation of the past and today.
(d) The author highlights the alarming degree to which misinformation has spread and how people have
lost their ability to resist misinformation.

111. Which among the following can be inferred from the passage?
(a) Our ability to resist misinformation in spite of corrective evidence may vary from time to time.
(b) The perception that technology has acted as a catalyst for misinformation is not free from errors.
(c) The New York Sun incident was one of the first incidents of misinformation to have surfaced.
(d) An important reason for why misinformation has so much endurance is repetition of fake news.

Head Office: 127, Zone II, MP Nagar, Bhopal |+91-7676564400| https://www.toprankers.com Page 30 of 40
112. Which one of the following, if true, presents a strong confirmation to the author’s arguments?
(a) Online media has given all the resources of deception to the peddlers of mistruth.
(b) The government has been able to mitigate the effects of misinformation.
(c) A dip in ability to resist misinformation does not render us susceptible to demagoguery.
(d) None of the above statements strengthen the author’s arguments.

113. If the information set out in the passage is true, which among the following must be true?
(a) Misinformation can be spread by technology as well as by humans.
(b) The spread of misinformation cannot be altered by anything or any phenomenon.
(c) Both a and b
(d) Neither a nor b

Passage(Q.114-Q.117): Often misunderstood, too much sitting is not synonymous with too little exercise.
In fact, a person considered highly active might sit far too much throughout their day: picture an individual
who lifts weights in the morning and runs for two hours in the evening, but sits in the car during his
commute and at a desk for seven hours throughout the workday. This individual certainly meets physical
activity guidelines but might still be at risk for detrimental health outcomes due to excessive sedentary
behavior. It appears that regardless of physical activity level, increased time sitting corresponds to an
increased risk of mortality.
Though the undeniable association between higher risk for disease and greater time spent sitting paints a
bleak picture, research investigating the manner in which sitting time is accumulated offers hope. In one
study, one to two minute breaks from sitting correlated with smaller waists, less insulin resistance, and
lower levels of inflammation. All of the aforementioned areas are risk factors for detrimental health
outcomes including cancer and metabolic disorders. Together, these reports hint that breaks in sedentary
time may become a primary method to protect against its adverse effects.

Despite the pervasive health risks associated with sitting and documented means to temper them, even
workplaces, where people accumulate most of their sedentary time, have enacted very few changes. On
the whole, employers do not brainstorm ways to lessen the time their employees spend sitting and
employees do not clamor for opportunities to escape the confines of their office chairs. Even hiring
replacement workers increase company costs, disrupt normal workplace flow, and raise stress levels.
Considering the substantial blow that ailing employees can deal to companies, it seems more likely that
this lack of workplace changes results from ignorance to company policy modifications. Then, it is
probable that considerable workplace changes could take place following heightened emphasis on the
sedentary behavior’s confounding health risks.

114. Which of the following is most supported by the author’s argument?


(a) Anyone who sits down for long will get cancer
(b) No one should sit down for more than an hour any given day
(c) Excessive sedentary behavior is impervious to regular exercise
(d) Some employer’s care about how much time their employees spend sitting down

115. Which of the following can be inferred from the passage?


(a) The impact of sedentary behavior can be controlled by frequent breaks
(b) The present system of working 8 hours a day needs to change
(c) Sitting down is worse when a person is getting some exercise during the day
(d) Both (a) & (b)

Head Office: 127, Zone II, MP Nagar, Bhopal |+91-7676564400| https://www.toprankers.com Page 31 of 40
116. Which of the following, if true, would seriously undermine the author’s argument?
(a) Majority of people that sit down in their day without a break are able to produce more output than
those who don’t
(b) Majority of people that sit down in their day without a break are uncomfortable with taking a break in
between
(c) Both (a) & (b)
(d) Neither (a) nor (b)

117. The statement “People who have a job that keeps them stationary are likely to die earlier” is:
(a) Probably True (b) Definitely True (c) Probably False (d) Definitely False

Passage(Q.118-Q.121): As the Chinese Communist Party (CCP) turned 100, its supreme leader and
China’s President-for-life Xi Jinping claimed his country had never bullied or subjugated any other nation
nor would it ever do so. But the message that rang across the world was exactly the opposite. The
reason is simple. Actions speak louder than words.

The CCP has much to be proud of and as much to be ashamed of. Millions of Chinese perished in the
Cultural Revolution and the Great Leap Forward and an unknown number bore the brunt of its fury in the
Tiananmen Square protests. Not to forget the millions of Uyghurs currently lodged in detention camps in
Xinjiang. On the other hand, the CCP uplifted millions of Chinese from poverty with its Open Doors Policy
or rather, the unnatural marriage of socialism with market economics —Socialism with Chinese
characteristics. The father of this peculiar formulation, Deng Xiaoping, had cautioned his comrades to
bide their time until the country became rich before flexing muscles on the international stage. Three
decades on, Xi surely believes the time has come. China has been clocking a 9.8% growth rate on an
average since 1979 and was the only country to register growth last year despite Covid. Evidently the
most powerful Chinese leader since Mao, Xi has arrogated to himself immense power by ensuring that
the two-term limit on presidency is removed, his own Xi Jinping Thought incorporated in the Constitution
and the party purged of his opponents.

His reign since 2012 has seen China becoming more assertive, be it in the South China Sea or the
borders with India, arm-twisting countries economically, upping the ante on Taiwan and refusing to be
transparent about Covid’s origins. Xi’s Belt and Road Initiative, promotion of authoritarian capitalism and
export of digital technology appear aimed at reshaping the world with China at the center stage. Drum-
beating nationalism at home and itching for confrontations abroad, whether Xi’s tenure spells instability in
the CCP remains to be seen. He is definitely in the process of achieving that abroad.
Extracted with edits from The New Indian Express:

118. Which among the following, if true, presents a strong confirmation to the author’s arguments?
i. The CCP has committed some war crimes inside their nation in the last three decades.
ii. The CCP has done some phenomenal work in getting millions of people out of poverty.
iii. The CCP last year forcefully interfered with the internal affairs of the nation Hong Kong.
(a) Both I and II (b) Both II and III (c) Both I and III (d) All I, II, and III

119. With which among the following views would the author of the passage be most likely to agree with?
(a) The claims of Xi Jinping after the CCP turned 100 has substance.
(b) The throne of Chinese Emperor Xi Jinping is at risk of being taken over.
(c) The time for China about which Deng Xiaoping talked about has come now.
(d) The Chinese Communist Party has not done anything phenomenal during its tenure.

Head Office: 127, Zone II, MP Nagar, Bhopal |+91-7676564400| https://www.toprankers.com Page 32 of 40
120. Which of the following is an assumption(s) based on which the author’s arguments depend?
(a) Unusual partnerships can sometimes yield positive results.
(b) An average growth rate of 9.8 % per annum for over three decades is uncommon for nations.
(c) Both (a) and (b) are invalid assumptions
(d) Neither (a) nor (b) are invalid assumptions

121. Which question has not been reasonably answered by the question in the passage?
(a) For what works should the Chinese Communist Party be proud of?
(b) What changes has the CCP gone after Xi Jinping came to power?
(c) What are the threats to the power wielded by Xi Jinping?
(d) All the above

Passage(Q.122-Q.126): Dumbledore: In modern day Cricket, full-out aggression is key. The previous
days’ “Gentleman’s Game” is gone. Take Virat Kohli for instance, he is known for his aggression and he
has captained his side to the number one ranking team in the world. At a time when the world is in a rat-
race for finishing first, the urge to give it back or to get under the skin of your opponent is the key. It’s a
dog-eat-dog world. It was due to Virat’s infused aggression that India could win the Australian Test Tour
even with Virat missing the last 3 of the 4 test matches.

Merlin: Oh yeah? If that is the case then why did India not win the first test match in which Virat was
captaining? They lost the test match then. See, a calm head is what sails you through dangerous waters.
I cite Dhoni and Ajinkya. They are calm in demeanor but aggressive in tactics. Just because you set up
an aggressive field for the opposing batsman, doesn’t mean that you have a license to have a go at the
opposing batsman. Making the correct changes and taking the right steps matter, aggressive or not. It
was no wonder that under Ajinkya, India won 2 out of those last 3 test matches.

122. What does Dumbledore mean when he says that “It’s a dog-eat-dog world”?
(a) The world of now is not that ethical. Winning is the key and for that aggression can be used.
(b) How people behave with us, we should behave with them. If someone is aggressive, we have to be
the same with them.
(c) Only aggression can be able to fight off aggression, otherwise it will overpower you.
(d) All of the above.

123. Which of the following, if true, is a flaw in Merlin’s argument about the first test match in which Virat was
captaining?
(a) Being aggressive in tactic means the person is aggressive. Even though the demeanor is cool, that
person will still be called aggressive.
(b) It was due to bowlers’ dismal performance that India lost the match and nothing else.
(c) Both (a) and (b)
(d) Neither (a) nor (b)

124. Which of the following is an assumption made by Dumbledore?


(a) Aggression was not a part of old days’ cricket
(b) Nowadays all the Cricket teams are aggressive
(c) Both (a) and (b)
(d) Neither (a) nor (b)

125. Which of the following is used by Dumbledore and Merlin to prove their arguments?
1. Examples 2. Idioms and phrases 3. Analogies
(a) Only 1 and 2 (b) Only 2 and 3 (c) Only 1 and 3 (d) None of the above

Head Office: 127, Zone II, MP Nagar, Bhopal |+91-7676564400| https://www.toprankers.com Page 33 of 40
126. Consider the following -
1. A marketing executive, to be successful, goes to the extent of contacting a competing firm’s
employee and brings out many insider information of its marketing strategy.
2. The marketing executive wants to be successful. He remains calm and does not lose his head when
he realizes his competition is way ahead of him. He calmly proceeds with the existing strategy.
3. Thor was beaten by Thanos in the Infinity Wars. However, in the Endgame Wars, he goes to battle
with Thanos again with the same strategy. But this time, he was less angry than the first time.
4. Iron-Man understands it will be difficult to beat Thanos, as the latter is much more powerful than him.
So instead of trying to fight him, he steals the infinity stones from Thanos – the source of the latter’s
power and then fights him.
Which of the above is a suitable analogy for Merlin’s and Dumbledore’s arguments?
(a) Dumbledore – 1 and 2; Merlin – 3 and 4
(b) Dumbledore – 1 and 3; Merlin – 2 and 4
(c) Dumbledore – 2; Merlin – 3
(d) Dumbledore – 1; Merlin – 4

Passage(Q.127-Q.131):UP’s population policy for 2021-30 unveiled by CM Yogi Adityanath wants to


incentivize couples to stick to a two-child norm. It’s an idea whose time has long gone. The past decade
has witnessed a sharp fall in India’s total fertility rates, even in UP, India’s most populous state. NFHS-4
data from 2015-16 indicates UP’s TFR fell to 2.7 in 2015-16 from 3.8 ten years prior. Sample Registration
System report for 2018 pegged UP’s TFR slightly higher at 2.9 in 2018 but even this was a fall of 23.1%
in a decade, bettering the national TFR decline of 18.5%.

Falling TFRs are massive demographic changes catalyzed by education of girls, economic growth,
migration, falling infant mortality rates and higher institutional births. All happened without drastic
population control policies. By all indications, UP will also hit replacement TFR levels of 2.1 in due course
like other states.

Apart from being unnecessary, the approach taken by the UP Law Commission’s draft Population
(Control, Stabilization and Welfare) Bill is also dangerous. It has prescriptions portending a bureaucratic
nightmare for ordinary citizens, especially the poor. It requires devoting energies to identifying state
employees and the general public who qualify for a raft of special incentives. Disincentives barring
access to welfare schemes and even the PDS for larger families, though not retrospective, poses
exclusion risks, massive corruption and social discrimination.

Badly conceived laws which trust bureaucracy with inordinate control over people’s lives are a recipe for
disaster. Ironically, the poor, and especially UP’s rural areas where TFR is 3.0 against 2.1 in cities, will be
penalized for structural deficiencies in schooling, public healthcare, and employment opportunities. The
potential for widespread social disruption can even undermine any political gains Adityanath may expect
in the UP assembly polls from the draft bill’s ample references to polygamy. UP should junk this idea.

Extracted from: The Times of India

127. What is the main conclusion drawn up by the author in the passage?
(a) The Uttar Pradesh government should junk its population policy for 2021-30.
(b) Incentivizing couples for having lesser children is a better population control measure than penalizing
them for having more children.
(c) The population policy of Uttar Pradesh government will not serve any meaningful purpose.
(d) Evidence suggests that women in India prefer having lesser children, if given a choice.

Head Office: 127, Zone II, MP Nagar, Bhopal |+91-7676564400| https://www.toprankers.com Page 34 of 40
128. Which among the following, if true, may weaken the author’s main argument in the passage above?
(a) UP’s population is already coming under control so the population policy of 2021-30 will not have any
effect.
(b) Reliable data suggests that India is not being threatened by a ‘population explosion’.
(c) Both a and b will not weaken the author’s main argument.
(d) Both a and b will weaken the author’s main argument.

129. Which among the following is an assumption made by the Uttar Pradesh government?
i. Penalizing couples for having more than two children will encourage them to have more children.
ii. Incentivizing couples for having less than two children will positively impact the state’s population
numbers.
iii. The population control measure will ensure that the state’s population of a particular community
declines.
(a) Both I and II (b) Both II and III
(c) All I, II, and III (d) None of the above

130. Which among the following is most likely to be inferred from the passage?
(a) Two-child norm might have been a reasonable measure for population control in the past.
(b) The poor population of Uttar Pradesh will be made to suffer for their past mistakes.
(c) Population control measures that may be dangerous are the way to go in the 21st century.
(d) Results of educating married women about population control have not been encouraging.

131. Which among the following has been used as a premise by the author to support his claims?
(a) The TFR of Uttar Pradesh was 3.8 in 2005-06.
(b) The TFR of Uttar Pradesh is projected to be at 1.7 in 2023-24.
(c) Retrospective effects of the population control measure may trigger massive social discrimination and
corruption.
(d) Both a and c

Passage(Q.132-Q.134): The Indian Medical Association (IMA) recently warned that a third wave of the
pandemic was inevitable and imminent and appealed to the Central and State governments not to let
down their guard against COVID-19. It urged them to not allow mass gatherings in tourist and religious
places without following the COVID-19 appropriate behavior.

In a letter to the Prime Minister, the IMA said that while tourist bonanza, pilgrimage travel and religious
fervor all were needed, they could wait for a few more months. According to it, opening up rituals and
enabling people without vaccination to go scot-free in such mass gatherings were potential super-
spreaders of a third wave.

“The past experience of last one and a half years of war with the virus and based on the emerging
evidences, it is obvious that by making the universal vaccination reach the maximum possible population
and strictly adopting to Covid appropriate behaviors, we can face the third wave with confidence and
mitigate its impact,’’ it stated. The IMA national president said, “It is the duty and responsibility of
everyone at this moment to strictly enforce the Covid appropriate behaviors for a minimum three more
months and ensure everyone gets vaccinated’’.

Extracted with edits and revisions from The Hindu

Head Office: 127, Zone II, MP Nagar, Bhopal |+91-7676564400| https://www.toprankers.com Page 35 of 40
132. Which among the following can be concluded to be true according to the passage?
(a) People who have been vaccinated would not contribute to the possibility of the third wave.
(b) The author agrees with the concerns of the Prime Minister regarding the onset of the third wave.
(c) The author believes that everyone should fulfill their duty of following Covid appropriate behavior at
this moment.
(d) None of the above

133. Based only on the information set out in the passage, with which among the following may be true?
i. Being complacent in following Covid-19 guidelines may invite the third wave.
ii. Pilgrimage travel and religious fervor should be postponed by a few months.
iii. The IMA is concerned about the possibility of a third wave.
(a) Only I and III (b) Only II and III (c) Only III (d) All I, II, and III

134. It is a mistake to see the electoral success of religious majoritarianism in recent years as constructed in a
vacuum. Instead, it is precisely the decades-long compartmentalization of different religious communities,
and the absence of solid state and civil society arrangements in educational pedagogy, personal
relationships, workplace, to facilitate inter-cultural interaction, and based on equality and respect, even
under supposedly secular regimes, that has made the soil politically fertile for the demonization of the
minority, especially the Muslims.
Which among the following best summarizes the main idea of the text?
(a) Perceiving that the electoral success of religious majoritarianism was constructed in a vacuum is
inaccurate.
(b) The demonization of the minority including Muslims should be stopped by analyzing the success of
the majority.
(c) Facilitating intercultural interaction presents one of the strongest reasons for the demonization of the
minority.
(d) Had there been the presence of civil society arrangements in various spheres of life, the minority
would not be demonized.

Head Office: 127, Zone II, MP Nagar, Bhopal |+91-7676564400| https://www.toprankers.com Page 36 of 40
SECTION - E :QUANTITATIVE TECHNIQUES

Directions (Q.135 – Q.139): Refer to the table given below and answer the given questions.

Data related to the number of employees in 5 different companies in December 2020

Out of total number of employees


Percentage
Percentage of
Total number of of
Company Percentage of Science graduates commerce
employees Arts
graduates
graduates
X - 30% 30% -
Y - - 40% 20%
Z - 35% 50% -
K 1000 32% - -
L 600 - 42% 30%

Note: Some values are missing, you have find out these value according to the question.
Note: Suppose that all the employees are graduated.

135. What is the difference between the number of commerce graduates employees and Arts graduates
employees in company L?
(a) 12 (b) 18 (c) 10 (d) 22

136. The average number of commerce graduates employees and science graduate employees in company Z
was 338. What was the total number of employees in company Z ?
(a) 1020 (b) 1140 (c) 1040 (d) 1240

137. If the respective ratio between the number of science graduate and commerce graduate employees in
company K was 10 : 7. What was the number of commerce graduate employees in K?
(a) 180 (b) 280 (c) 380 (d) 80

138. Total number of employees in company L increased by 20% from December 2020 to December 2021. If
20% of the total number of employees in company L in December 2021 was Arts graduate, what was the
number of Arts graduate employees in company L in December 2021?
(a) 144 (b) 169 (c) 244 (d) 104

139. Total number of employees in company X was three time the total number of employees in company Y. If
the difference between number of commerce graduate employees in company Y and that of science
graduate employees in same company was 120, what was the total number of employees in company X?
(a) 600d (b) 1200 (c) 1800 (d) 3000

USE FOR ROUGH WORK

Head Office: 127, Zone II, MP Nagar, Bhopal |+91-7676564400| https://www.toprankers.com Page 37 of 40
Directions(Q.140 – Q.144): In Organization A, Ratio of female and male is 6:7. Female at Organization
C is 750 more than Male at Organization A. In Organization B, Male and Female are equal. Male at D is
250 more than the double of Female at A. Female at D is 2550.Male and Female ratio at D is 11:15. Total
Male are 5655. And Male at C is double the Female at A.

140. 20% Female of D are doing MBA and 25% Female of B are doing MBA. What is total no. of
Female doing MBA from these organizations?
(a) 655 (b) 743 (c) 815 (d) 735

141. What is the average (approx.) Male in all the organizations together?
(a) 1414 (b) 5655 (c) 5234 (d) 4567

142. Out of the Total people doing job at A in three departments, marketing, accounts, and IT are in
the ratio of 4:3:2 respectively. Then total people in IT department?
(a) 234 (b) 450 (c) 760 (d) 390

143. Age of some people of every organization is below 30 and some are above 30. 12.5% of total employee
from B is above 30 and25% of total employee of D is above 30. Then find the ratio of above 30 employee
of D to the same of B?
(a) 123:67 (b) 221:61 (c) 223:61 (d) 234:67

144. 20% out of total Female and 25 % out of total Male at C has resigned. What is total number of people
who are still working at C?
(a) 2571 (b) 3456 (c) 3456 (d) 453

USE FOR ROUGH WORK

Head Office: 127, Zone II, MP Nagar, Bhopal |+91-7676564400| https://www.toprankers.com Page 38 of 40
Directions(Q.145 – Q.147): Answer the questions based on the information given below.

Out of 200 people who attended a marriage party 100 had GolGape, 120 had Chicken Tica and 40 had
none of these two. 100 people had exactly one of two items.

145. How many people like had both of the given items?
(a) 35 (b) 50 (c) 55 (d) 60

146. How many person does not like Chiken Tica?


(a) 55 (b) 80 (c) 35 (d) 50

147. Person who like only Golgape is what % more or less than the person who like only chicken Tica?
(a) 38.5% (b) 25.6% (c) 15.2% (d) 33.33%

Directions(Q.148 – Q.150): Read the passage carefully and answer the following questions.
X Females can do a work in 2Y days. 1.5X men can do a work in Y days. 8 males, 8 Females and 8
children together can do the work in 22.5 days. 3X child can do the work in 4Y days.

148. If 2.5X females can complete the work in 24 days. Then Find the value of Y?
(a) 35 (b) 37 (c) 29 (d) 30

149. Find in how many days 10 males and 12 females together can complete the same work.
(a) 19(31/38) (b) 17(31/38) (c) 19(29/38) (d) 17(29/38)

150. How many days 16 females complete the whole work.


(a) 27(5/8) (b) 31(7/38) (c) 31(1/8) (d) 28(1/8)

USE FOR ROUGH WORK

Head Office: 127, Zone II, MP Nagar, Bhopal |+91-7676564400| https://www.toprankers.com Page 39 of 40
Notes:

Head Office: 127, Zone II, MP Nagar, Bhopal |+91-7676564400| https://www.toprankers.com Page 40 of 40

You might also like